Ask your own question, for FREE!
Mathematics 53 Online
OpenStudy (loser66):

Show that if \(c_1, c_2,\cdots, c_{\phi (m)}\) is a reduced residue system modulo m, where m is a positive integer with \(m \neq 2\), then \(c_1+c_2+\cdots+c_{\phi (m)}=0 modm\) Please, help

OpenStudy (ikram002p):

so no two elements in \(c_1, c_2,\cdots, c_{\phi (m)}\) have same modulo of m but yet each one have some modulo of m, right ?

OpenStudy (loser66):

I can see it in small case, but don't know how to generalize it. See!! if m =10, then reduced residue modulo 10 is {1,3,7, 9}, then 1+3+7+9 =20 and \(20\equiv 0(mod 10)\) but how to argue with m case?

OpenStudy (ikram002p):

well read my first comment which means in some order we can rewrite the set as :- \(c_1, c_2,\cdots, c_{\phi (m)} =(1,2,3..., m-1 \mod m)\)

OpenStudy (ikram002p):

agree to this ?

OpenStudy (loser66):

Nope :) since m is a positive integer, we don't know what it is yet, if it is an even number, then 2 is not in the set. Am I right?

OpenStudy (ikram002p):

wait sorry let me read it again =)

ganeshie8 (ganeshie8):

Hints : 1) \(\phi(m)\) is even for \(m\gt 2\) 2) If \(c_1\) is in the residue set, then \(m-c_1\) is also in the residue set.

OpenStudy (loser66):

Got you @ganeshie8 and I got the answer also. :)

Can't find your answer? Make a FREE account and ask your own questions, OR help others and earn volunteer hours!

Join our real-time social learning platform and learn together with your friends!
Can't find your answer? Make a FREE account and ask your own questions, OR help others and earn volunteer hours!

Join our real-time social learning platform and learn together with your friends!